« first day (1749 days earlier)      last day (3289 days later) » 

3:00 PM
we need an excisive triad in the general case
 
i'll believe you. i haven't done homology theory at the general level.
don't see the point of doing so.
 
me neither :/
 
Hey @BalarkaSen really sorry but I didn't get anything from your hint
Can you please elaborate a little more
 
@Remember $S = \{z : d(z, 0) \leq 2\}$ be your set, $B_\epsilon((2, 0))$ be a small ball around $(2, 0)$.
 
Yes fine
 
3:04 PM
You want to prove that $B_\epsilon((2, 0)) \cap S$ is nonempty, right?
 
Yes
You said try using triangle inequality how should I do it
 
eh, I don't think you need triangle ineq.
You can do it by hand.
Pick $(2 - \epsilon', 0)$, where $0 < \epsilon' < \epsilon$.
 
Okay
 
Verify that this is inside the intersection.
 
hi @Balarka @Remember bananas
 
3:10 PM
Well the first set contains all points which are less than or equal to 2 since it is 2 minus something which is less than two and hence in the S
 
no ping for @iwriteonbananas, lol
 
Is topology your main interest?
 
morning @Ted
 
@BalarkaSen?
 
g'night, @Balarka :)
 
3:11 PM
nah. but I like it.
 
I can see that.
What is, then?
 
@TedShifrin Mike has retired, then?
 
Morning @TedShifrin
 
or did you want more participants to play that game with? :P
 
Balarka is the proof right
 
3:12 PM
'salright
@octatoan My main interesting is topological topology.
In general, mathematical mathematics.
 
user147690
Balarka is talking nonsense again hahaha
 
?
 
Ah got it
 
I am just trying to pull feeble jokes, @octatoan
 
Pj's
 
3:14 PM
Right now, I am interested in topology, but interests change.
 
You like funny fun, then.
Ah.
 
mornin', @Mike.
 
His main study is commutative abelian rings with identity...
 
hi @AlexC
oh, goodnight, @MikeM
 
user147690
Hi @TedS
 
3:16 PM
I was thinking about replying "no, it's finitely venerated abelian poops" but thought better of it.
 
well, actually, @Balarka, isn't it getting to be night there?
 
Okay, so here Rudin talks about $x>0$ and then sets $t=\frac{x}{x+1}$. How is $t\geq0$?
 
Yah those are too well understood
no one would believe you
 
Shouldn't he have written $t<0$ instead?
 
Surely it's $t\ge 0$, @octatoan.
 
3:17 PM
@PaulPlummer What, abelian poops?
 
Although the other thing is true too
Ah, sorry.
Wrong way.
 
Yes, sure, of course, $t>0$.
 
Hm.
Thanks.
 
Well finitely venerated venerated abelian poops @BalarkaSen
 
"What's purple, commutes, and is worshipped by a limited number of people? A finitely-venerated Abelian grape."
This is the one I know . . .
 
3:20 PM
I hadn't heard the limitedly worshipped version :P
 
Q: What is lavender and commutes?
A: An Abelian semigrape.
 
ok, enough.
 
lol, @Ted
 
morning @Ted
 
I guess he did retire
 
3:23 PM
I just threw out my qualifying exam prep folders from 1974-75, @MikeM. I thought about scanning you some of the topology questions, but they're beneath you now. :P
 
in the mayer vietoris sequence, the map $H_n(X_1) \oplus H_n(X_2) \to H_n(X)$ is $i_* + j_*$, where i,j are the inclusions, right?
 
You should have sent some of them to me, @Ted :( But I'll find them online anyways.
yeah, @iwriteonbananas
 
Don't be silly, @Ted, they hadn't invented topology back then.
 
No you won't, @Balarka. I made up all sorts of concrete alg top/diff top questions.
I'll tell Rob Kirby you said so, @MikeM. He was one of my examiners.
 
3:24 PM
ah, you mean it was made by you.
I see.
 
user147690
@octatoan Hahahahahaha
 
What?
 
Click the little arrow
 
user147690
@octatoan I am so tired that I found that really funny
 
user147690
On that note I should sleep
 
user147690
3:26 PM
Ta ta
 
Oh. G'night Alex, ta ta :)
 
See yah
 
Night, @AlexC.
 
See yah
 
Surprisingly, @Ted, Kirby will not be in my mathematical genealogy, despite his overarching influence.
 
3:27 PM
No question of the latter.
 
Morally we are all Kirby's children.
 
ps, @Mike, did you see what I pinged you about homology of infinite cyclic cover of knot complements and cech homology of "knotted" solenoidal spaces?
d'you think anything can be done with it?
 
No to the former.
 
okay.
 
@MikeMiller Not I.
 
3:30 PM
Fair enough, son of Chern
 
I wonder if something more general can be said about homology of universal cover of spaces and cech homology of solenoidal spaces obtained from taking inverse limit of all the (branched) covers.
 
I don't know what those words mean.
 
which word, in particular?
 
"Solenoidal space". Do you just mean an inverse limit of spaces?
 
yeah, inverse limit of covering spaces with bonding maps being covering maps.
 
3:36 PM
OK, I read the thing you linked. I don't see much reason it should have anything to do with the Alexander polynomial and I vehemently disagree that Cech homology is easier to handle.
It behaves well for a wider class of spaces, but that does not make it easy for a human being to handle.
 
well, ok.
I guess I am just interested as it relates to the kind of things I like to think about :P
 
Sure.
 
@BalarkaSen do you know any category theory?
 
a bit
 
@TedShifrin hello, Are these two norms equivalent : $||u||_{L^p}+||\nabla u||_{L^p}$ and $||u||^p_{L^p}+||\nabla u||^p_{L^p}$ please thank you
 
3:46 PM
Guys simple question: Is a lattice graph of an arbitrary number of sides n, always possible? I know triangular, square, pentagonal and hexagonal lattice graphs exist along with all polygons that can be constructed from a combination of them. But what about polygons with a prime number of sides? Can a lattice graph still be constructed in that case? I don't need a proof, just a confirmation.
 
@Vrouvrou: The second isn't a norm, is it?
 
Rudin chapter 1 done.
Winter is coming.
Does anyone know of any normal-looking non-archimedean subsets of $\mathbb{R}$?
Of the same cardinality?
 
what's a non-archimedean subset
 
A subset $S$ for which it is not true that there always exists integer $n>0$ such that $nx>y$ for $x,y \in S$ and $x, y > 0$
It's true for $\mathbb{R}$ and it's called the archimedean property.
 
I don't understand. there are no non-archimedian subsets of $\Bbb R$. do you mean such that $nx$ is also in $S$?
 
4:00 PM
@TedShifrin right i'm sorry the norm on W^{1,p} is $||u||_{L^p}+||\nabla u||_{L^p}$ and not $||u||^p_{L^p}+||\nabla u||^p_{L^p}$
 
Yes.
 
gotcha
 
Is there such a subset?
 
If you require $nx \in S$ then singetons are pretty normal looking
 
"Of the same cardinality"
Yes
 
4:01 PM
Of the same cardinality of $\mathbb R$?
 
I wouldn't be surprised if such subsets existed; I wouldn't be surprised if it was independent of ZFC
 
Yes
 
@TedShifrin please see this if you have an idea math.stackexchange.com/questions/1284540/a-counter-example
 
i would be a little surprised if you could actually prove non-existence
 
Hm.
Just wondering aloud, is all.
Back to work.
 
4:02 PM
Nah, pretty sure it shouldn't be that difficult
 
well, actually, no, you're going to want something like density
$[0,1]$ doesn't satisfy it eg
 
Subtract all rational multiples of $\pi$, except $\pi$ from $\Bbb R$, then take $y=4$
 
@TedShifrin i want to find for example a sequence which converge in $C_{\theta}$ but not in $L^{p^*}_1$
 
oh
meh
just need that integral multiples don't exist
 
Density.
 
4:05 PM
that's dense
 
I've heard that word a lot.
 
True,
 
@Vrouvrou: I don't think enough about this sort of stuff to be helpful, sorry. I'm working on cleaning out my office now.
 
I think the above example has every property you could possibly want
 
Yeah.
Thanks.
 
4:06 PM
@TedShifrin ok , do you know someone who can help me ?
 
Or you can just consider $\Bbb Q \cup \{ \pi \}$, I guess
(square root 2, anything that is not rational
 
@Vrouvrou: That's why you post on Main. You get all sorts of people who might be able to respond.
 
Or consider $\Bbb R / \Bbb Q$, choose one element from each coset, so that it ends up dense (this uses choice). This is an instance of $G_1 (D,D)$, where $D$ is every dense subset of $\Bbb R$ :) @MikeMiller
Then you have no two elements elements have this "archimedian" property @octatoan
even $x,x$ since $nx$ is not in the set when $n>1$
(well it is an instance of the game if you choose from only countably many, but you can extend the game to continuum many innings)
 
4:24 PM
I don't know that much group theory yet. :(
 
@TedShifrin i'm sorry but can you explain me this in french :Your first space contains only continuous functions by definitions while your second doesn't. Take u to be the indicator of some subset of Ω and you have your counterexample
 
u is the counterexample
@Vrouvrou
does that help?
 
You don't need group theory, this is a quotient construction and it basically says that $x,y$ are in the same coset(equivalence class) if and only if $x-y$ is rational. @octatoan
 
Oh, this was an exercise in the first chapter of Aluffi!
 
That sounds about right
 
4:29 PM
Yeah, about equivalence classes and everything. Thanks.
 
@octatoan i juste don't clearly understand it
 
Although you should change "don't know much group theory"... :D
 
I haven't done all of Aluffi.
.Just the first chapter.
 
I know, just saying you should learn more group theory (although just learn whatever you want)
 
I'll do Artin now, then Aluffi.
Yeah, I like algebra.
 
4:30 PM
please +1 for this question if i have only -1 i will can't ask a question again :mathoverflow.net/questions/207047/…
 
@Vrouvrou that is not a research level question I think
 
Is that research level? I am not going to upvote a question that should not be on MO
 
Looks like an exercise.
 
no it is a research level question there are new spaces and there is no this in papers
 
Oh, well
 
4:33 PM
@robjohn You here?
 
I'm more concerned about why the integral doesn't have the curl at the top on MO
(No pun intended.)
 
@octatoan the person who gives me the counter example "Hachino " delete it because it is not good
 
Okay, but this is getting sort of spammy now.
 
it means that this question is not easy
 
r9m
@Chris'ssis okay ,, I have a way .. no gamma functions nothing ... 1st year calculus and induction :)
3
 
4:36 PM
i deleted it on Mo i recived -2
pfff
 
"spammy"
 
@r9m Great. :-) I think I also have a pretty elementary way.
 
why "spammy" i don't understand @octatoan ?
 
@r9m that is using $2p$th derivative of the (cosh(x))^n at the point $x=0$.
 
r9m
@Chris'ssis EXACTLY SAME APPROACH we had then :D
 
4:41 PM
@r9m :D:D:D:D:D:D:D
@r9m Take care about Knuth's problem, it is wrong. I corrected it and sent it to AMM. They accepted my solution. :-)
@r9m the story with the Catalan number generating function is wrong.
 
r9m
@Chris'ssis wow! okay! :)
 
@r9m One page solution. (and 2 tiny rows I think one the other page):-)
 
r9m
@Chris'ssis omg! Nice :)
 
@r9m :D
@r9m Don't forget to ask me to show you my solution in case they don't publish it (hope it's not that case).
 
r9m
@Chris'ssis sure! :-) I wait eagerly :D
 
4:46 PM
(after they publish the solutions to this problem)
 
Hello @r9m
Remember me?
 
r9m
@Rememberme hi
 
@r9m you are still in CMI?
 
r9m
@Rememberme yes
 
So where are you going to do your MSC@r9m
 
r9m
4:52 PM
@Rememberme Tifr Bangalore
 
@octatoan you do -1 for my question so give me an idea !
 
You beauty!!! TIFR
So we might hope to meet some day :D@r9m
 
Hi @cirpis @DanielFischer There are given 3 vectors v_1, v_2, v_3, of dimension m, of which the elements are real numbers.
I tried to write a quicker than $O(m^2)$ algorithm that finds if there are three numbers , one from each of the vectors v_1, v_2, v_3, that have sum equal to 0.
Is the following idea right?
We look at the elements that are at the midpoint of our interval. If v_1[mid]+v_2[mid]+v_3[mid]=0, we are done. If v_1[mid]+v_2[mid]+v_3[mid]>0, we decrement the index of the last vector by one. If v_1[mid]+v_2[mid]+v_3[mid-1]>0, we decrement the index of the seond vector by 1. If v_1[mid]+v_2[mid-1]+v_3[mid-1]>0, we decrement the index of the first vector by 1. If v_1[mid-1]+v_2[mid-1]+v_3[mid-1]>0, we decrement the index of the third vector by 1 and so on.
And similarly if v_1[mid]+v_2[mid]+v_3[mid]<0.
 
r9m
@Rememberme sure :)
 
Well where is tifr in blore I don't know the address sadly@r9m
 
4:55 PM
@Vrouvrou I didn't.
 
r9m
@Rememberme It's in Yelahanka Satellite Town
 
Ahh yelahanka pretty far away from my place a 2 hour drive from my house @r9m
Your course is in pure maths or applied maths @r9m
 
r9m
@Rememberme pure math .. mainly focusing on mathematical analysis
 
Ahh.... Topology also??
Nice analysis you surely going to have fun
 
r9m
@Rememberme the main focus is on functional analysis and partial differential equations
 
5:01 PM
@Chris'ssis Can you compute $\sum_{n=1}^\infty\frac{(-1)^n}{n+1}H_n$ ?
 
@RickDecker Hi!!! Do you maybe have an idea about the following?

There are given 3 vectors v_1, v_2, v_3, of dimension m, of which the elements are real numbers.
I tried to write a quicker than $O(m^2)$ algorithm that finds if there are three numbers , one from each of the vectors v_1, v_2, v_3, that have sum equal to 0.
Is the following idea right?

We look at the elements that are at the midpoint of our interval. If v_1[mid]+v_2[mid]+v_3[mid]=0, we are done. If v_1[mid]+v_2[mid]+v_3[mid]>0, we decrement the index of the last vector by one. If v_1[mid]+v_2[mid]+v_3[mid-1]>0, we decrement
 
@LeGrandDODOM elementary. First arrange a bit the numerator and write $H_{n+1}-1/(n+1)$. Then use Cauchy product, cleverly. Q.E.D.
 
@Chris'ssis tell me more
 
Getting a -1 does not mean it is easy and just because it may be hard does not mean it is research level
@Vrouvrou
 
@PaulPlummer it is a research level
 
5:04 PM
@LeGrandDODOM No need for that arrangement. See here, just Cauchy product math.stackexchange.com/questions/292973/…
@LeGrandDODOM I discovered it my research independently, but I knew it also appeared there.
 
How do you know a counter example exists? @Vrouvrou
 
@LeGrandDODOM you should also see @robjohn's way. Not sure now where that link is.
 
hi guys
 
@KarimMansour Hi
 
I upvoted your math.se question, maybe it will get more attention, but I don't know enough to say if it is a good fit for math.se, I have a feeling it is not, but idk @Vrouvrou
 
5:07 PM
@Chris'ssis are you thinking of this answer?
 
@robjohn Yes, precisely! :-)
 
because i take a space $L^{p^*}_{\alpha}$ and i foud a condition on $\alpha$ to have a continuous embedding from C_{\theta} to L^{p^*}_{\alpha} it was $\alpha >1$ so i take $\alpha=1$ @PaulPlummer
 
@LeGrandDODOM see the link above robjohn just showed.
 
hi @Karim salut @LeGrandDODO
hi @robjohn
 
@TedShifrin good morning/afternoon.
 
5:09 PM
LOL ... something like that, @robjohn :)
 
Morning @Ted, @robjohn
 
salut @Ted.
 
We've been through this already, @MikeM. I said goodnight hours ago :P
 
Was I there for that?
 
ta ta @TedShifrin as the British, and the Indians seem to say
 
5:11 PM
How would I know, Mike? :D
 
@MikeMiller I was at UCLA for a midterm on Friday. I was pretty tight-scheduled, and on the way home, a got a flat tire.
 
oh no, @robjohn
I always worry that something like that will happen to me on the way to the airport at 6 AM
 
robjohn grad student?
 
@robjohn Ah, that's awful.
 
@KarimMansour nah... I was a grad student over 30 years ago.
 
5:13 PM
he's just young compared to me, @Karim
 
Anyone here good at German and reading Fraktur?
 
@Ted: I remember now.
 
oh I see must be a prof or something
 
No one is good at reading Fraktur :D
 
Who would want to read much fraktur?
 
5:14 PM
The only German I know is "Arsch ersetzer".
 
Artin's first chapter.
 
Hey @robjohn
Did you see my question?
 
oh, his little quote?
 
Yes.
 
I don't have the book here, though.
 
5:14 PM
isn't that ass hole or something @MikeMiller?
 
It's packed somewhere.
 
No, @Karim, don't be crass.
 
Endlich wird alles . . . ?
I know a little German, it won't suffice for all of that.
 
yeah, mine either at this point
 
I learned german but I don't like it
 
5:15 PM
DanielF is a German speaker.
 
weird language
 
^ I went to Germany last year for a month for a school program. When I came back everyone was curious why I was ending my sentences with prepositions.
 
5:31 PM
@Chris'ssis well that Cauchy product is non standard for none of the series converges absolutely
 
@LeGrandDODOM You find the needed theorem in the Joseph Edwards's old book such that you don't need to be concerned at all.
 
@octatoan No, you should have ended your sentences with past participles.
 
Haha.
Instant Yoda?
 
@LeGrandDODOM There are many ways to get there. For instance you can use the generating function of the harmonic numbers combined with a bit of integration and Taylor series ...
 
I want to learn like 3 languages more I already know 3 :D
 
5:44 PM
Duolingo is your friend.
 
Hey everyone, quick question. When you have two vector spaces V and W, what's the standard way to denote the set of all linear maps V -> W?
 
hey @TedShifrin
so I was just looking at this example in DF
1 moment typing
 
@DavidZ: $\text{Hom}(V,W)$ or $L(V,W)$.
 
@TedShifrin Is it usually a plain italic $L$, not a script or double-struck one?
 
@Ted: $P$ is a complex polynomial. Assume $|P(z)| \leq M$ for all $z$ in the unit circle. What's an example of one such that $M < \sum |a_i|$, where these are the coefficients?
 
5:51 PM
It can be whatever you want, @DavidZ. Some people use each of these.
 
We show that the center center of $D_8$ is subgroup ${1,r^2}$ we know that $r^3$ isn't in the center well they didn't show it but I did the following computation $sr^3 = r^{-3}s = r^{-2}r^{-1}s = r^5s = rs \neq sr^5 = sr$ but there is other way to show this easier way I guess
I can feel it in my gut
 
Nevermind. $(z-1/2)^2$.
 
I was about to say it shouldn't be hard, @MikeM, but I don't have one off the top of my head.
 
The set of untimely June insects is dense in my room.
 
No, that doesn't work.
 
5:52 PM
what do you think @TedShifrin?
 
@Ted: It shouldn't be hard. A student claimed this on a honework, and I should be able to give a counterexample!!
 
Ah, I got into a very hard question trying to counterprove a homework claim in my grad complex analysis years ago.
Are we allowed to be geometric, @Karim?
 
@TedS: Surely there's an easy example...
 
this is for my reading @TedShifrin not for a class or anything just trying to make my mind think in multiple ways.
 
5:54 PM
Note that $r$ is conjugate to $r^3$, either by algebraic definition or by geometry, @Karim. But they're not equal.
 
@MikeMiller Just to make sure I understand what you're asking, $M$ is the least upper bound of $|P(z)|$ for $|z| < 1$, right?
 
Or the max on $|z|=1$, same thing.
 
Yes.
 
oh I see
 
@MikeM: Not swearing by this. But what about $1+iz-z^2$?
 

« first day (1749 days earlier)      last day (3289 days later) »